How to prove by mathematical induction that $(y-x)x^n leq frac{y^{n+1}-x^{n+1}}{n+1} leq (y-x)y^n$? [closed] ...

How did passengers keep warm on sail ships?

Can the DM override racial traits?

Are spiders unable to hurt humans, especially very small spiders?

My body leaves; my core can stay

Do I have Disadvantage attacking with an off-hand weapon?

What's the point in a preamp?

Student Loan from years ago pops up and is taking my salary

Single author papers against my advisor's will?

Is it ethical to upload a automatically generated paper to a non peer-reviewed site as part of a larger research?

How do spell lists change if the party levels up without taking a long rest?

Why doesn't a hydraulic lever violate conservation of energy?

Windows 10: How to Lock (not sleep) laptop on lid close?

Loose spokes after only a few rides

What information about me do stores get via my credit card?

Separating matrix elements by lines

Using dividends to reduce short term capital gains?

Is there a way to generate uniformly distributed points on a sphere from a fixed amount of random real numbers per point?

Working through the single responsibility principle (SRP) in Python when calls are expensive

Can the Right Ascension and Argument of Perigee of a spacecraft's orbit keep varying by themselves with time?

Accepted by European university, rejected by all American ones I applied to? Possible reasons?

Python - Fishing Simulator

Button changing its text & action. Good or terrible?

How do you keep chess fun when your opponent constantly beats you?

Does Parliament need to approve the new Brexit delay to 31 October 2019?



How to prove by mathematical induction that $(y-x)x^n leq frac{y^{n+1}-x^{n+1}}{n+1} leq (y-x)y^n$? [closed]



The 2019 Stack Overflow Developer Survey Results Are In
Unicorn Meta Zoo #1: Why another podcast?
Announcing the arrival of Valued Associate #679: Cesar ManaraProve $ sum_{1leq k < n} k^{underline{m}}=frac{n^{underline{m+1}}}{m+1} $ by induction on $m$Mathematical inductionUse induction to prove that $n! leq n^{n-1}$prove by induction that $F(n) leq left(frac{1 + sqrt{5}}{2}right)^n$Why is Mathematical Induction used to prove solvable inequalities?Prove the equality using complete mathematical induction.Proof that $ln(n+1) leq ln(n)+1$Proof by Mathematical Induction for InequalityHow to use mathematical induction to verify: $sum_{i=1}^{n}frac{1}{i(i+1)} = frac{n}{n+1}$Prove that $e^x cos (sqrt{x^2+1}) leq 1$












0












$begingroup$



Prove by induction that:
$$(y-x)x^n leq frac{y^{n+1}-x^{n+1}}{n+1} leq (y-x)y^n .
$$






As a hint, the professor told us to use the following expression that we had previously proven:
$$sum_{i=0}^n{x^i}= frac{1-x^{n+1}}{1-x}
$$



I already tried several things, but I can’t manage to get to the solution.










share|cite|improve this question











$endgroup$



closed as off-topic by Saad, John Omielan, user21820, RRL, Alexander Gruber Mar 24 at 2:54


This question appears to be off-topic. The users who voted to close gave this specific reason:


  • "This question is missing context or other details: Please provide additional context, which ideally explains why the question is relevant to you and our community. Some forms of context include: background and motivation, relevant definitions, source, possible strategies, your current progress, why the question is interesting or important, etc." – Saad, John Omielan, user21820, RRL, Alexander Gruber

If this question can be reworded to fit the rules in the help center, please edit the question.












  • 1




    $begingroup$
    I suppose you assume that $y>x$?
    $endgroup$
    – Yanko
    Mar 22 at 13:33










  • $begingroup$
    It definitely holds for $n=0$.
    $endgroup$
    – Wuestenfux
    Mar 22 at 13:34










  • $begingroup$
    Yes sorry. It’s for 0<x<y
    $endgroup$
    – Facu50196
    Mar 22 at 13:34










  • $begingroup$
    And what exactly have you tried?
    $endgroup$
    – Saad
    Mar 22 at 13:35










  • $begingroup$
    Were you really told to use induction and to use the result you had previously proved (presumably by using induction)? I can see no need to use both. You could use induction, but this would probably duplicate the proof of the previous result. You can use the previous result directly (i.e. without using induction again) by observing that if $x < 1$, then the left hand side is less than $n+1$ and greater than $(n+1)x^n$.
    $endgroup$
    – Calum Gilhooley
    Mar 23 at 15:37
















0












$begingroup$



Prove by induction that:
$$(y-x)x^n leq frac{y^{n+1}-x^{n+1}}{n+1} leq (y-x)y^n .
$$






As a hint, the professor told us to use the following expression that we had previously proven:
$$sum_{i=0}^n{x^i}= frac{1-x^{n+1}}{1-x}
$$



I already tried several things, but I can’t manage to get to the solution.










share|cite|improve this question











$endgroup$



closed as off-topic by Saad, John Omielan, user21820, RRL, Alexander Gruber Mar 24 at 2:54


This question appears to be off-topic. The users who voted to close gave this specific reason:


  • "This question is missing context or other details: Please provide additional context, which ideally explains why the question is relevant to you and our community. Some forms of context include: background and motivation, relevant definitions, source, possible strategies, your current progress, why the question is interesting or important, etc." – Saad, John Omielan, user21820, RRL, Alexander Gruber

If this question can be reworded to fit the rules in the help center, please edit the question.












  • 1




    $begingroup$
    I suppose you assume that $y>x$?
    $endgroup$
    – Yanko
    Mar 22 at 13:33










  • $begingroup$
    It definitely holds for $n=0$.
    $endgroup$
    – Wuestenfux
    Mar 22 at 13:34










  • $begingroup$
    Yes sorry. It’s for 0<x<y
    $endgroup$
    – Facu50196
    Mar 22 at 13:34










  • $begingroup$
    And what exactly have you tried?
    $endgroup$
    – Saad
    Mar 22 at 13:35










  • $begingroup$
    Were you really told to use induction and to use the result you had previously proved (presumably by using induction)? I can see no need to use both. You could use induction, but this would probably duplicate the proof of the previous result. You can use the previous result directly (i.e. without using induction again) by observing that if $x < 1$, then the left hand side is less than $n+1$ and greater than $(n+1)x^n$.
    $endgroup$
    – Calum Gilhooley
    Mar 23 at 15:37














0












0








0


1



$begingroup$



Prove by induction that:
$$(y-x)x^n leq frac{y^{n+1}-x^{n+1}}{n+1} leq (y-x)y^n .
$$






As a hint, the professor told us to use the following expression that we had previously proven:
$$sum_{i=0}^n{x^i}= frac{1-x^{n+1}}{1-x}
$$



I already tried several things, but I can’t manage to get to the solution.










share|cite|improve this question











$endgroup$





Prove by induction that:
$$(y-x)x^n leq frac{y^{n+1}-x^{n+1}}{n+1} leq (y-x)y^n .
$$






As a hint, the professor told us to use the following expression that we had previously proven:
$$sum_{i=0}^n{x^i}= frac{1-x^{n+1}}{1-x}
$$



I already tried several things, but I can’t manage to get to the solution.







inequality induction






share|cite|improve this question















share|cite|improve this question













share|cite|improve this question




share|cite|improve this question








edited Mar 23 at 14:04









Jack

27.7k1784204




27.7k1784204










asked Mar 22 at 13:30









Facu50196Facu50196

396




396




closed as off-topic by Saad, John Omielan, user21820, RRL, Alexander Gruber Mar 24 at 2:54


This question appears to be off-topic. The users who voted to close gave this specific reason:


  • "This question is missing context or other details: Please provide additional context, which ideally explains why the question is relevant to you and our community. Some forms of context include: background and motivation, relevant definitions, source, possible strategies, your current progress, why the question is interesting or important, etc." – Saad, John Omielan, user21820, RRL, Alexander Gruber

If this question can be reworded to fit the rules in the help center, please edit the question.







closed as off-topic by Saad, John Omielan, user21820, RRL, Alexander Gruber Mar 24 at 2:54


This question appears to be off-topic. The users who voted to close gave this specific reason:


  • "This question is missing context or other details: Please provide additional context, which ideally explains why the question is relevant to you and our community. Some forms of context include: background and motivation, relevant definitions, source, possible strategies, your current progress, why the question is interesting or important, etc." – Saad, John Omielan, user21820, RRL, Alexander Gruber

If this question can be reworded to fit the rules in the help center, please edit the question.








  • 1




    $begingroup$
    I suppose you assume that $y>x$?
    $endgroup$
    – Yanko
    Mar 22 at 13:33










  • $begingroup$
    It definitely holds for $n=0$.
    $endgroup$
    – Wuestenfux
    Mar 22 at 13:34










  • $begingroup$
    Yes sorry. It’s for 0<x<y
    $endgroup$
    – Facu50196
    Mar 22 at 13:34










  • $begingroup$
    And what exactly have you tried?
    $endgroup$
    – Saad
    Mar 22 at 13:35










  • $begingroup$
    Were you really told to use induction and to use the result you had previously proved (presumably by using induction)? I can see no need to use both. You could use induction, but this would probably duplicate the proof of the previous result. You can use the previous result directly (i.e. without using induction again) by observing that if $x < 1$, then the left hand side is less than $n+1$ and greater than $(n+1)x^n$.
    $endgroup$
    – Calum Gilhooley
    Mar 23 at 15:37














  • 1




    $begingroup$
    I suppose you assume that $y>x$?
    $endgroup$
    – Yanko
    Mar 22 at 13:33










  • $begingroup$
    It definitely holds for $n=0$.
    $endgroup$
    – Wuestenfux
    Mar 22 at 13:34










  • $begingroup$
    Yes sorry. It’s for 0<x<y
    $endgroup$
    – Facu50196
    Mar 22 at 13:34










  • $begingroup$
    And what exactly have you tried?
    $endgroup$
    – Saad
    Mar 22 at 13:35










  • $begingroup$
    Were you really told to use induction and to use the result you had previously proved (presumably by using induction)? I can see no need to use both. You could use induction, but this would probably duplicate the proof of the previous result. You can use the previous result directly (i.e. without using induction again) by observing that if $x < 1$, then the left hand side is less than $n+1$ and greater than $(n+1)x^n$.
    $endgroup$
    – Calum Gilhooley
    Mar 23 at 15:37








1




1




$begingroup$
I suppose you assume that $y>x$?
$endgroup$
– Yanko
Mar 22 at 13:33




$begingroup$
I suppose you assume that $y>x$?
$endgroup$
– Yanko
Mar 22 at 13:33












$begingroup$
It definitely holds for $n=0$.
$endgroup$
– Wuestenfux
Mar 22 at 13:34




$begingroup$
It definitely holds for $n=0$.
$endgroup$
– Wuestenfux
Mar 22 at 13:34












$begingroup$
Yes sorry. It’s for 0<x<y
$endgroup$
– Facu50196
Mar 22 at 13:34




$begingroup$
Yes sorry. It’s for 0<x<y
$endgroup$
– Facu50196
Mar 22 at 13:34












$begingroup$
And what exactly have you tried?
$endgroup$
– Saad
Mar 22 at 13:35




$begingroup$
And what exactly have you tried?
$endgroup$
– Saad
Mar 22 at 13:35












$begingroup$
Were you really told to use induction and to use the result you had previously proved (presumably by using induction)? I can see no need to use both. You could use induction, but this would probably duplicate the proof of the previous result. You can use the previous result directly (i.e. without using induction again) by observing that if $x < 1$, then the left hand side is less than $n+1$ and greater than $(n+1)x^n$.
$endgroup$
– Calum Gilhooley
Mar 23 at 15:37




$begingroup$
Were you really told to use induction and to use the result you had previously proved (presumably by using induction)? I can see no need to use both. You could use induction, but this would probably duplicate the proof of the previous result. You can use the previous result directly (i.e. without using induction again) by observing that if $x < 1$, then the left hand side is less than $n+1$ and greater than $(n+1)x^n$.
$endgroup$
– Calum Gilhooley
Mar 23 at 15:37










1 Answer
1






active

oldest

votes


















1












$begingroup$

I'm not quit sure how to use the hint given by the professor. But here's one way to do this:



Recall that $$(y^{n+1}-x^{n+1}) = (y-x)(y^n+y^{n-1}x+....+yx^{n-1}+x^{n})$$



Now assuming that $x<y$ you can replace all the $y$'s with $x$'s in the second multiple1 and get that $$y^{n+1}-x^{n+1} geq (y-x) (n+1)x^n$$ Divide by $n+1$ and you get the first inequality. For the other one replace the $x$'s with $y$'s.





1.Edit It's possible that you need to prove this part by induction. That is, prove by induction that $$(n+1)x^n leq y^n+y^{n-1}x+...+yx^{n-1}+x^n$$






share|cite|improve this answer











$endgroup$













  • $begingroup$
    Thanks!! I’ll try that
    $endgroup$
    – Facu50196
    Mar 22 at 13:58


















1 Answer
1






active

oldest

votes








1 Answer
1






active

oldest

votes









active

oldest

votes






active

oldest

votes









1












$begingroup$

I'm not quit sure how to use the hint given by the professor. But here's one way to do this:



Recall that $$(y^{n+1}-x^{n+1}) = (y-x)(y^n+y^{n-1}x+....+yx^{n-1}+x^{n})$$



Now assuming that $x<y$ you can replace all the $y$'s with $x$'s in the second multiple1 and get that $$y^{n+1}-x^{n+1} geq (y-x) (n+1)x^n$$ Divide by $n+1$ and you get the first inequality. For the other one replace the $x$'s with $y$'s.





1.Edit It's possible that you need to prove this part by induction. That is, prove by induction that $$(n+1)x^n leq y^n+y^{n-1}x+...+yx^{n-1}+x^n$$






share|cite|improve this answer











$endgroup$













  • $begingroup$
    Thanks!! I’ll try that
    $endgroup$
    – Facu50196
    Mar 22 at 13:58
















1












$begingroup$

I'm not quit sure how to use the hint given by the professor. But here's one way to do this:



Recall that $$(y^{n+1}-x^{n+1}) = (y-x)(y^n+y^{n-1}x+....+yx^{n-1}+x^{n})$$



Now assuming that $x<y$ you can replace all the $y$'s with $x$'s in the second multiple1 and get that $$y^{n+1}-x^{n+1} geq (y-x) (n+1)x^n$$ Divide by $n+1$ and you get the first inequality. For the other one replace the $x$'s with $y$'s.





1.Edit It's possible that you need to prove this part by induction. That is, prove by induction that $$(n+1)x^n leq y^n+y^{n-1}x+...+yx^{n-1}+x^n$$






share|cite|improve this answer











$endgroup$













  • $begingroup$
    Thanks!! I’ll try that
    $endgroup$
    – Facu50196
    Mar 22 at 13:58














1












1








1





$begingroup$

I'm not quit sure how to use the hint given by the professor. But here's one way to do this:



Recall that $$(y^{n+1}-x^{n+1}) = (y-x)(y^n+y^{n-1}x+....+yx^{n-1}+x^{n})$$



Now assuming that $x<y$ you can replace all the $y$'s with $x$'s in the second multiple1 and get that $$y^{n+1}-x^{n+1} geq (y-x) (n+1)x^n$$ Divide by $n+1$ and you get the first inequality. For the other one replace the $x$'s with $y$'s.





1.Edit It's possible that you need to prove this part by induction. That is, prove by induction that $$(n+1)x^n leq y^n+y^{n-1}x+...+yx^{n-1}+x^n$$






share|cite|improve this answer











$endgroup$



I'm not quit sure how to use the hint given by the professor. But here's one way to do this:



Recall that $$(y^{n+1}-x^{n+1}) = (y-x)(y^n+y^{n-1}x+....+yx^{n-1}+x^{n})$$



Now assuming that $x<y$ you can replace all the $y$'s with $x$'s in the second multiple1 and get that $$y^{n+1}-x^{n+1} geq (y-x) (n+1)x^n$$ Divide by $n+1$ and you get the first inequality. For the other one replace the $x$'s with $y$'s.





1.Edit It's possible that you need to prove this part by induction. That is, prove by induction that $$(n+1)x^n leq y^n+y^{n-1}x+...+yx^{n-1}+x^n$$







share|cite|improve this answer














share|cite|improve this answer



share|cite|improve this answer








edited Mar 23 at 12:34

























answered Mar 22 at 13:35









YankoYanko

8,4692830




8,4692830












  • $begingroup$
    Thanks!! I’ll try that
    $endgroup$
    – Facu50196
    Mar 22 at 13:58


















  • $begingroup$
    Thanks!! I’ll try that
    $endgroup$
    – Facu50196
    Mar 22 at 13:58
















$begingroup$
Thanks!! I’ll try that
$endgroup$
– Facu50196
Mar 22 at 13:58




$begingroup$
Thanks!! I’ll try that
$endgroup$
– Facu50196
Mar 22 at 13:58



Popular posts from this blog

Magento 2 - Add success message with knockout Planned maintenance scheduled April 23, 2019 at 23:30 UTC (7:30pm US/Eastern) Announcing the arrival of Valued Associate #679: Cesar Manara Unicorn Meta Zoo #1: Why another podcast?Success / Error message on ajax request$.widget is not a function when loading a homepage after add custom jQuery on custom themeHow can bind jQuery to current document in Magento 2 When template load by ajaxRedirect page using plugin in Magento 2Magento 2 - Update quantity and totals of cart page without page reload?Magento 2: Quote data not loaded on knockout checkoutMagento 2 : I need to change add to cart success message after adding product into cart through pluginMagento 2.2.5 How to add additional products to cart from new checkout step?Magento 2 Add error/success message with knockoutCan't validate Post Code on checkout page

Fil:Tokke komm.svg

Where did Arya get these scars? Unicorn Meta Zoo #1: Why another podcast? Announcing the arrival of Valued Associate #679: Cesar Manara Favourite questions and answers from the 1st quarter of 2019Why did Arya refuse to end it?Has the pronunciation of Arya Stark's name changed?Has Arya forgiven people?Why did Arya Stark lose her vision?Why can Arya still use the faces?Has the Narrow Sea become narrower?Does Arya Stark know how to make poisons outside of the House of Black and White?Why did Nymeria leave Arya?Why did Arya not kill the Lannister soldiers she encountered in the Riverlands?What is the current canonical age of Sansa, Bran and Arya Stark?